LSAT and Law School Admissions Forum

Get expert LSAT preparation and law school admissions advice from PowerScore Test Preparation.

 Administrator
PowerScore Staff
  • PowerScore Staff
  • Posts: 8916
  • Joined: Feb 02, 2011
|
#23057
Complete Question Explanation

StrengthenX-CE. The correct answer choice is (A)

Why has the reported asthma death rate doubled? The author offers two possible explanations: Perhaps the underlying actual deaths have not changed but the reporting has become more accurate. Alternatively, maybe pollution is responsible for an increase in actual deaths. But the author dismisses both of these explanations by showing that even where neither causal force was present, reported asthma death rates have still increased. Finally, the author offers a third explanation: the use of bronchial inhalers. The author's conclusion seems like a huge logical leap: Why is this third explanation compelling? The author provides no evidence. Also, the author does not examine whether any other explanations might be salient.

Read the question stem carefully. Four of the answer choices will support the conclusion, but one answer choice — the correct answer choice — will not.

Answer choice (A): This is the correct answer choice, as it fails to support the author's argument. First of all, it does nothing to support the "inhalers" explanation. Second, it fails to undermine the "urban pollution" explanation. If the urban asthma death rate is greater than the non-urban asthma death rate (and especially if the gap is rising due to increased pollution), then a relative population shift from non-urban areas to urban areas would increase the overall asthma death rate.

Answer choice (B): According to this answer choice, asthma death reporting has not actually become more accurate. This is a devastating blow to the possible explanation about more accurate reporting. This supports the author's position.

Answer choice (C): This answer choice provides evidence showing why bronchial inhalers might lead to death from asthma. This supports the author's position.

Answer choice (D): This answer choice provides evidence showing why use of bronchial inhalers might increase the risk of death from asthma. This supports the author's position.

Answer choice (E): This answer choice demonstrates that bronchial inhalers are a new causal force, not present ten years ago. This supports the author's position. (If the statement in this answer choice were not true, then the "inhalers" explanation would not be compelling. In that case, why were the inhalers not causing deaths before ten years ago?)
 deck1134
  • Posts: 160
  • Joined: Jun 11, 2018
|
#48225
Hi PowerScore Staff,

I really do not understand this question.

We want an answer that does NOT support the argument. If more people move to cities, doesn't that explain why more people die? I don't understand your "urban population" explanation as given in above.
User avatar
 Dave Killoran
PowerScore Staff
  • PowerScore Staff
  • Posts: 5853
  • Joined: Mar 25, 2011
|
#48892
deck1134 wrote:Hi PowerScore Staff,

I really do not understand this question.

We want an answer that does NOT support the argument. If more people move to cities, doesn't that explain why more people die? I don't understand your "urban population" explanation as given in above.
No, it doesn't :-D Look closely at the stimulus: "death rate" is being discussed. Thus, more people moving there—which is an actual number—doesn't change death rates.

This is a good example of how to review problems as well. We know (A) is right, so instead of focusing on your interpretation of the problem and why an answer is right or wrong, focus on LSAC's interpretation. Why do they think it's right or wrong? You are entering their world on the test, and so your goal is to get into their heads and understand how they think. It will help you tremendously in terms of scoring!!

Thanks!
 deck1134
  • Posts: 160
  • Joined: Jun 11, 2018
|
#48896
Thanks, Dave! That is definitely something that I am working on!
 lolaSur
  • Posts: 46
  • Joined: Nov 11, 2019
|
#72409
Now that I understand that "death rate" is defined differently from "number of deaths," I see how Answer A does not support the argument. However, I still do not understand how Answer B supports the conclusion that the cause of increased deaths is the use of bronchial inhalers by asthma sufferers to relieve their symptoms.

Could you please explain a little further how Answer B supports the argument?

Thank you so much.



For my reference (L5, A/J/S q41).
 James Finch
PowerScore Staff
  • PowerScore Staff
  • Posts: 943
  • Joined: Sep 06, 2017
|
#72428
Hi Lola,

The stimulus gives us three possible causes for the effect of increased reporting of deaths of asthma sufferers in the past 10 years: better records, more pollution, or inhalers, and ultimately decides that inhalers are the cause. (B) tells us that our records are equally good if we go back 20 years or only 10, which helps rule out the alternate cause of record keeping being responsible for the increase in the numbers of reported deaths in the last 10 years. Ruling out an alternate cause strengthens a causal conclusion, so (B) strengthens the conclusion here.

Hope this clears things up!
 blade21cn
  • Posts: 100
  • Joined: May 21, 2019
|
#82857
A quick question. It seems to me that "(B) records of asthma deaths are as accurate for the past twenty years as for the past ten years" merely contradicts the opposing premise "the recording of deaths due to asthma has become more widespread and accurate in the past decade than it had been previously," which, in the LSAT world, must be taken as true. Is there another way to explain how (B) strengthens the argument, which is the link between the premise and the conclusion? Thanks!
 Adam Tyson
PowerScore Staff
  • PowerScore Staff
  • Posts: 5153
  • Joined: Apr 14, 2011
|
#84256
That statement in the stimulus is not a premise, blade21cn - it's listed as a possible explanation that is later ruled out by the author. That possible explanation has been offered by others, not by the author, and the author is telling us that it cannot be the correct explanation. Answer B supports the premise that better records cannot be the explanation, rather than contradicting a premise.

Get the most out of your LSAT Prep Plus subscription.

Analyze and track your performance with our Testing and Analytics Package.